Finding Supremum and Infimum of Sets with Inequalities

In summary: For $n = 1$, $1+\frac{1}{n}=2$, so $2\leq 1+\frac{1}{n}$ and so $1+\frac{1}{n}$ is not the supremum.For $n = 2$, $1+\frac{1}{n}=3$, so $3\leq 1+\frac{1}{n}$ and so $1+\frac{1}{n}$ is not the supremum.For $n = 3$, $1+\frac{1}{n}=4$, so $4\
  • #1
evinda
Gold Member
MHB
3,836
0
Hello! (Wave)

I want to find the supremum, infimum of the following sets:

$$\{ x \in \mathbb{R}: 0<x^2-1<3\}, \{1+\frac{(-1)^n}{n}: n=1,2, \dots \}$$

For the first set I have thought the following:

$$ 0<x^2-1<3 \Rightarrow 1<x^2<4 \Rightarrow x^2>1 \text{ and } x^2 <4 \Rightarrow (x>1 \text{ or } x<-1) \text{ and } -2<x<2 \Rightarrow (-2<x<-1) \text{ or } (1<x<2)$$

Can we say that $-2$ is the infimum of the set and $2$ the supremum? Or not, since only one of these two inequalities hold? (Thinking)

For the second set, I have thought the following:$1+\frac{(-1)^n}{n}=\left\{\begin{matrix}
1+\frac{1}{n}, &\text{ if n is even} \\ \\
1-\frac{1}{n}, & \text{ if n is odd.}
\end{matrix}\right.$

We have that for all $n \in \mathbb{N}$, $1+\frac{1}{n} \geq 1$ and $1+\frac{1}{n} \leq 2$.

Also, we have that $1-\frac{1}{n} \leq 1$ and $1-\frac{1}{n} \geq 0$.

So is $0$ the infimum and $2$ the supremum? (Thinking)
 
Physics news on Phys.org
  • #2
evinda said:
Hello! (Wave)

I want to find the supremum, infimum of the following sets:

$$\{ x \in \mathbb{R}: 0<x^2-1<3\}, \{1+\frac{(-1)^n}{n}: n=1,2, \dots \}$$

For the first set I have thought the following:

$$ 0<x^2-1<3 \Rightarrow 1<x^2<4 \Rightarrow x^2>1 \text{ and } x^2 <4 \Rightarrow (x>1 \text{ or } x<-1) \text{ and } -2<x<2 \Rightarrow (-2<x<-1) \text{ or } (1<x<2)$$

Can we say that $-2$ is the infimum of the set and $2$ the supremum? Or not, since only one of these two inequalities hold? (Thinking)

You can replace your $\Rightarrow$ with $\Leftrightarrow$ because you are actually describing the set in a number of ways that are all equivalent. Yes, you are correct. (In this case, it may also help to draw the graph of the standard parabola and see for which values of $x$ it lies strictly between $1$ and $4$.)
evinda said:
For the second set, I have thought the following:$1+\frac{(-1)^n}{n}=\left\{\begin{matrix}
1+\frac{1}{n}, &\text{ if n is even} \\ \\
1-\frac{1}{n}, & \text{ if n is odd.}
\end{matrix}\right.$

We have that for all $n \in \mathbb{N}$, $1+\frac{1}{n} \geq 1$ and $1+\frac{1}{n} \leq 2$.

Also, we have that $1-\frac{1}{n} \leq 1$ and $1-\frac{1}{n} \geq 0$.

So is $0$ the infimum and $2$ the supremum? (Thinking)

I agree with the inf but not with the sup.
 
  • #3
Janssens said:
You can replace your $\Rightarrow$ with $\Leftrightarrow$ because you are actually describing the set in a number of ways that are all equivalent. Yes, you are correct. (In this case, it may also help to draw the graph of the standard parabola and see for which values of $x$ it lies strictly between $1$ and $4$.)

Looking at this again, since the infimum is the greatest lower bound for the set, shouldn't it be equal to $1$ ?

More specifically, the lower bounds of the set are $1,-2$ and since $1$ is greater, this is the infimum.

Similarly, the upper bounds are $-1$ and $2$ and the smaller of them is $-1$. Thus, this is the supremum.

Or am I wrong? (Thinking)
 
  • #4
evinda said:
Looking at this again, since the infimum is the greatest lower bound for the set, shouldn't it be equal to $1$ ?

More specifically, the lower bounds of the set are $1,-2$ and since $1$ is greater, this is the infimum.

Similarly, the upper bounds are $-1$ and $2$ and the smaller of them is $-1$. Thus, this is the supremum.

Or am I wrong? (Thinking)

Now you are wrong and initially you were right. (Wink)

In interval notation, the set in question is: $(-2, -1) \cup (1, 2)$.
A lower bound has to be a lower bound for the whole set (i.e. the union of the two open intervals), not just for one of the intervals.
So, $-\pi$ is a lower bound, as is $-2$, but $1$ is not a lower bound.
For the upper bounds a similar comment applies.
 
  • #5
Janssens said:
I agree with the inf but not with the sup.

Why isn't $2$ the supremum? (Thinking)

- - - Updated - - -

Janssens said:
Now you are wrong and initially you were right. (Wink)

In interval notation, the set in question is: $(-2, -1) \cup (1, 2)$.
A lower bound has to be a lower bound for the whole set (i.e. the union of the two open intervals), not just for one of the intervals.
So, $-\pi$ is a lower bound, as is $-2$, but $1$ is not a lower bound.
For the upper bounds a similar comment applies.

Ah I see... (Nod)
 
  • #6
Janssens said:
I agree with the inf but not with the sup.

We have that $n \geq 1$ and so $\frac{1}{n} \leq 1 \Rightarrow 1+\frac{1}{n} \leq 2$. So why isn't $2$ the supremum? (Thinking)
 
  • #7
evinda said:
We have that $n \geq 1$ and so $\frac{1}{n} \leq 1 \Rightarrow 1+\frac{1}{n} \leq 2$. So why isn't $2$ the supremum? (Thinking)

It is an upper bound, yes, but not the sup.
Try writing down the first few elements of the set, say for $n = 1, 2, 3, 4$.
(Don't bother simplifying, just leave the terms as $1 \pm \frac{1}{n}$.)
 
  • #8
Janssens said:
It is an upper bound, yes, but not the sup.
Try writing down the first few elements of the set, say for $n = 1, 2, 3, 4$.
(Don't bother simplifying, just leave the terms as $1 \pm \frac{1}{n}$.)

We have that $1 \pm \frac{1}{n} \leq 1+\frac{1}{n}$.

We notice that $1+\frac{1}{n} \to 1$, while $n \to +\infty$.

Thus $\sup\{1+\frac{(-1)^n}{n}: n=1,2, \dots \}=1$.

Right? (Thinking)
 
Last edited:
  • #9
evinda said:
We have that $1 \pm \frac{1}{n} \leq 1+\frac{1}{n}$.

We notice that $1+\frac{1}{n} \to 1$, while $n \to +\infty$.

Thus $\sup\{1+\frac{(-1)^n}{n}: n=1,2, \dots \}=1$.

Right? (Thinking)

No, go back for a moment. $1$ is not an upper bound, so it cannot be the sup.
What value occurs for $n=2$?
What other values occur? Do any of these exceed the value for $n = 2$?
 
  • #10
Janssens said:
No, go back for a moment. $1$ is not an upper bound, so it cannot be the sup.
What value occurs for $n=2$?
What other values occur? Do any of these exceed the value for $n = 2$?

For $n=1$ we get that $1+\frac{1}{n}=2$, for $n=2$ we get that $1+\frac{1}{n}=1.5$, for $n=3$ we get that $1+\frac{1}{n}=1.333$ and for $n=4$ we get that $1+\frac{1}{n}=1.25$.

Since $\frac{1}{n}$ is a decreasing function, we will always get smaller values, won't we?

Why isn't the supremum the minimum value of $1+\frac{1}{n}$, which we get when $n \to +\infty$? (Wondering)
 
  • #11
evinda said:
For $n=1$ we get that $1+\frac{1}{n}=2$, for $n=2$ we get that $1+\frac{1}{n}=1.5$, for $n=3$ we get that $1+\frac{1}{n}=1.333$ and for $n=4$ we get that $1+\frac{1}{n}=1.25$.

Since $\frac{1}{n}$ is a decreasing function, we will always get smaller values, won't we?

Why isn't the supremum the minimum value of $1+\frac{1}{n}$, which we get when $n \to +\infty$? (Wondering)

Careful, for $n = 1$ we get $1 + \frac{(-1)^1}{1} = 0$. Generally, for odd $n$ we subtract $\frac{1}{n}$ and for even $n$ we add $\frac{1}{n}$.
Also, the supremum must be an upper bound. The set looks like
$$
\left\{0, 1 + \frac{1}{2}, 1 - \frac{1}{3}, 1 + \frac{1}{4}, \ldots.\right\},
$$
and while the associated sequence indeed converges to $1$, it is clear that $1$ is not an upper bound for the set.
 
  • #12
Janssens said:
Careful, for $n = 1$ we get $1 + \frac{(-1)^1}{1} = 0$. Generally, for odd $n$ we subtract $\frac{1}{n}$ and for even $n$ we add $\frac{1}{n}$.
Also, the supremum must be an upper bound. The set looks like
$$
\left\{0, 1 + \frac{1}{2}, 1 - \frac{1}{3}, 1 + \frac{1}{4}, \ldots.\right\},
$$
and while the associated sequence indeed converges to $1$, it is clear that $1$ is not an upper bound for the set.

Ah I see... But then how can we find the least upper bound? (Thinking)
 
  • #13
evinda said:
Ah I see... But then how can we find the least upper bound? (Thinking)

I am sure you can do that.

What would be a good candidate for the sup? Consider how the set looks in my previous post.
The candidate is a member of the set.

Once found, prove that the candidate is an upper bound.
Since it is also a member of the set, there cannot be a smaller upper bound.
 
  • #14
Janssens said:
I am sure you can do that.

What would be a good candidate for the sup? Consider how the set looks in my previous post.
The candidate is a member of the set.

Once found, prove that the candidate is an upper bound.
Since it is also a member of the set, there cannot be a smaller upper bound.

We have that $1 \pm \frac{1}{n} \leq 1+\frac{1}{n}\leq 1+\frac{1}{2}=\frac{3}{2}$.

Thus $\frac{3}{2}$ is the supremum of the set. Right? (Thinking)
 
  • #15
evinda said:
We have that $1 \pm \frac{1}{n} \leq 1+\frac{1}{n}\leq 1+\frac{1}{2}=\frac{3}{2}$.

Thus $\frac{3}{2}$ is the supremum of the set. Right? (Thinking)

Right!
 
  • #16
Janssens said:
Right!

Nice, thank you! (Happy)
 

What is the definition of supremum and infimum of a set?

The supremum of a set is the smallest upper bound, or the least upper bound, of the set. In other words, it is the smallest number that is greater than or equal to every element in the set. The infimum of a set is the largest lower bound, or the greatest lower bound, of the set. This means that it is the largest number that is less than or equal to every element in the set.

How do you find the supremum and infimum of a set?

To find the supremum of a set, you must first determine if the set has an upper bound. If it does not, then the supremum does not exist. If the set does have an upper bound, you can find the supremum by taking the maximum value of the set. To find the infimum, you must determine if the set has a lower bound. If not, the infimum does not exist. If the set does have a lower bound, you can find the infimum by taking the minimum value of the set.

What is the difference between supremum and maximum?

The supremum of a set may or may not be an element of the set itself, whereas the maximum is always an element of the set. This means that the supremum can be thought of as the smallest possible upper bound, while the maximum is the largest element in the set.

Can the supremum and infimum of a set be equal?

Yes, the supremum and infimum of a set can be equal. This happens when the set has only one element, or when the set is a closed interval with both endpoints included. In other cases, the supremum and infimum will be different.

How are supremum and infimum related to each other?

The supremum and infimum are related by duality, meaning that the roles of the upper and lower bounds are reversed. This means that if a number is the supremum of a set, it is also the infimum of the set's inverse, and vice versa. Additionally, the supremum and infimum are always equal in a totally ordered set, where every element can be compared to every other element.

Similar threads

  • Topology and Analysis
Replies
4
Views
2K
  • Topology and Analysis
Replies
2
Views
1K
  • Topology and Analysis
Replies
4
Views
277
  • Topology and Analysis
Replies
9
Views
1K
Replies
1
Views
203
  • Topology and Analysis
Replies
21
Views
1K
Replies
1
Views
713
Replies
2
Views
1K
Replies
13
Views
1K
  • Topology and Analysis
2
Replies
38
Views
3K
Back
Top